Proving a three variables inequality











up vote
3
down vote

favorite












Given that $frac{1}{a}+frac{1}{b}+frac{1}{c}=1$ show that:$$(a+1)(b+1)(c+1)ge 64$$
My attempt: First I tried expanding the LHS getting that$$abc+ab+bc+ca+a+b+c ge 63$$
I applied Cauchy-Schwarz on $(a+b+c)(frac{1}{a}+frac{1}{b}+frac{1}{c})$ getting that $a+b+cge9$.



Then I also tried to manipulate the first condition and got $abc=ab+bc+ca$, then I applied AM-GM on $a+b+c$ getting the following$$a+b+cge3sqrt[3]{abc}$$
Finally I substituted $ab+bc+ca=abc$ in my initial expression, getting:$$2abc+(a+b+c)ge63$$The last thing I tought about was that I have both $a+b+cge9$ and $a+b+cge3sqrt[3]{abc}$ so if I somehow related them I would have $sqrt[3]{abc} ge 3 rightarrow abcge27$ and with this conditions the problem would follow by summing, but the direction of the inequality is not allowing me to do as intended...



I'm stuck here, have tried lot of other things but nothing really worked, also partial help is appreciated!










share|cite|improve this question






















  • I haven't tried this I admit .... but my instant thought is, since we 'know' (please note the apostrophæ!) that it attains its minimum value at a=b=c=3, could we not hypothesise that it's 3, plug in 3+δ, 3+ε, & 3+ζ, & show that any increase in some measure of the size of δ, ε & ζ causes an increase in the product under the constraint? As I said - just my instantaneous thought on seeing the question!
    – AmbretteOrrisey
    24 mins ago















up vote
3
down vote

favorite












Given that $frac{1}{a}+frac{1}{b}+frac{1}{c}=1$ show that:$$(a+1)(b+1)(c+1)ge 64$$
My attempt: First I tried expanding the LHS getting that$$abc+ab+bc+ca+a+b+c ge 63$$
I applied Cauchy-Schwarz on $(a+b+c)(frac{1}{a}+frac{1}{b}+frac{1}{c})$ getting that $a+b+cge9$.



Then I also tried to manipulate the first condition and got $abc=ab+bc+ca$, then I applied AM-GM on $a+b+c$ getting the following$$a+b+cge3sqrt[3]{abc}$$
Finally I substituted $ab+bc+ca=abc$ in my initial expression, getting:$$2abc+(a+b+c)ge63$$The last thing I tought about was that I have both $a+b+cge9$ and $a+b+cge3sqrt[3]{abc}$ so if I somehow related them I would have $sqrt[3]{abc} ge 3 rightarrow abcge27$ and with this conditions the problem would follow by summing, but the direction of the inequality is not allowing me to do as intended...



I'm stuck here, have tried lot of other things but nothing really worked, also partial help is appreciated!










share|cite|improve this question






















  • I haven't tried this I admit .... but my instant thought is, since we 'know' (please note the apostrophæ!) that it attains its minimum value at a=b=c=3, could we not hypothesise that it's 3, plug in 3+δ, 3+ε, & 3+ζ, & show that any increase in some measure of the size of δ, ε & ζ causes an increase in the product under the constraint? As I said - just my instantaneous thought on seeing the question!
    – AmbretteOrrisey
    24 mins ago













up vote
3
down vote

favorite









up vote
3
down vote

favorite











Given that $frac{1}{a}+frac{1}{b}+frac{1}{c}=1$ show that:$$(a+1)(b+1)(c+1)ge 64$$
My attempt: First I tried expanding the LHS getting that$$abc+ab+bc+ca+a+b+c ge 63$$
I applied Cauchy-Schwarz on $(a+b+c)(frac{1}{a}+frac{1}{b}+frac{1}{c})$ getting that $a+b+cge9$.



Then I also tried to manipulate the first condition and got $abc=ab+bc+ca$, then I applied AM-GM on $a+b+c$ getting the following$$a+b+cge3sqrt[3]{abc}$$
Finally I substituted $ab+bc+ca=abc$ in my initial expression, getting:$$2abc+(a+b+c)ge63$$The last thing I tought about was that I have both $a+b+cge9$ and $a+b+cge3sqrt[3]{abc}$ so if I somehow related them I would have $sqrt[3]{abc} ge 3 rightarrow abcge27$ and with this conditions the problem would follow by summing, but the direction of the inequality is not allowing me to do as intended...



I'm stuck here, have tried lot of other things but nothing really worked, also partial help is appreciated!










share|cite|improve this question













Given that $frac{1}{a}+frac{1}{b}+frac{1}{c}=1$ show that:$$(a+1)(b+1)(c+1)ge 64$$
My attempt: First I tried expanding the LHS getting that$$abc+ab+bc+ca+a+b+c ge 63$$
I applied Cauchy-Schwarz on $(a+b+c)(frac{1}{a}+frac{1}{b}+frac{1}{c})$ getting that $a+b+cge9$.



Then I also tried to manipulate the first condition and got $abc=ab+bc+ca$, then I applied AM-GM on $a+b+c$ getting the following$$a+b+cge3sqrt[3]{abc}$$
Finally I substituted $ab+bc+ca=abc$ in my initial expression, getting:$$2abc+(a+b+c)ge63$$The last thing I tought about was that I have both $a+b+cge9$ and $a+b+cge3sqrt[3]{abc}$ so if I somehow related them I would have $sqrt[3]{abc} ge 3 rightarrow abcge27$ and with this conditions the problem would follow by summing, but the direction of the inequality is not allowing me to do as intended...



I'm stuck here, have tried lot of other things but nothing really worked, also partial help is appreciated!







inequality






share|cite|improve this question













share|cite|improve this question











share|cite|improve this question




share|cite|improve this question










asked 2 hours ago









Spasoje Durovic

1218




1218












  • I haven't tried this I admit .... but my instant thought is, since we 'know' (please note the apostrophæ!) that it attains its minimum value at a=b=c=3, could we not hypothesise that it's 3, plug in 3+δ, 3+ε, & 3+ζ, & show that any increase in some measure of the size of δ, ε & ζ causes an increase in the product under the constraint? As I said - just my instantaneous thought on seeing the question!
    – AmbretteOrrisey
    24 mins ago


















  • I haven't tried this I admit .... but my instant thought is, since we 'know' (please note the apostrophæ!) that it attains its minimum value at a=b=c=3, could we not hypothesise that it's 3, plug in 3+δ, 3+ε, & 3+ζ, & show that any increase in some measure of the size of δ, ε & ζ causes an increase in the product under the constraint? As I said - just my instantaneous thought on seeing the question!
    – AmbretteOrrisey
    24 mins ago
















I haven't tried this I admit .... but my instant thought is, since we 'know' (please note the apostrophæ!) that it attains its minimum value at a=b=c=3, could we not hypothesise that it's 3, plug in 3+δ, 3+ε, & 3+ζ, & show that any increase in some measure of the size of δ, ε & ζ causes an increase in the product under the constraint? As I said - just my instantaneous thought on seeing the question!
– AmbretteOrrisey
24 mins ago




I haven't tried this I admit .... but my instant thought is, since we 'know' (please note the apostrophæ!) that it attains its minimum value at a=b=c=3, could we not hypothesise that it's 3, plug in 3+δ, 3+ε, & 3+ζ, & show that any increase in some measure of the size of δ, ε & ζ causes an increase in the product under the constraint? As I said - just my instantaneous thought on seeing the question!
– AmbretteOrrisey
24 mins ago










4 Answers
4






active

oldest

votes

















up vote
7
down vote



accepted










By the super-additivity of the geometric mean / convexity of $log(e^x+1)$
$$ (1+a)(1+b)(1+c) geq (1+sqrt[3]{abc})^3 $$
and by the GM-HM inequality
$$ sqrt[3]{abc} geq frac{3}{frac{1}{a}+frac{1}{b}+frac{1}{c}}=3.$$






share|cite|improve this answer

















  • 1




    Please explain the downvote. I do not see anything wrong with this approach, it is pretty straightforward.
    – Jack D'Aurizio
    2 hours ago






  • 3




    Maybe the downvoter's arithmetic is so bad that they want you explain that $4^3 = 64$ $ddot{smile}$.
    – Rob Arthan
    2 hours ago








  • 1




    @JackD'Aurizio I'm convinced that there are users who stays here on MSE only to downvote, probably to give vent to frustration in the real world!
    – gimusi
    2 hours ago








  • 1




    @greedoid Nothing personal with the comment, I was only joking of course.
    – gimusi
    2 hours ago






  • 1




    @RobArthan Not a theory but only a joke! Bye :)
    – gimusi
    1 hour ago


















up vote
5
down vote













I think you mean $$a,b,c>0$$ in this case we have
$$frac{a+b+c}{3}geq frac{3}{frac{1}{a}+frac{1}{b}+frac{1}{c}}$$ so $$a+b+cgeq 9$$ and we get also $$frac{bc+ac+ab}{3}geq sqrt[3]{(abc)^2}$$ so $$abcgeq 27$$ and $$ab+ac+bcgeq 27$$ putting things together we have
$$(a+1)(b+1)(c+1)=abc+ab+bc+ca+a+b+c+1geq 27+27+10=64$$






share|cite|improve this answer





















  • Welcome back Dr. Graubner!
    – gimusi
    1 hour ago










  • Yeah, OP forgot to mention that $a,b,c$ have to be positive, I forgot to add that I had noticed that
    – Spasoje Durovic
    1 hour ago










  • Thanks, i'm also glad to see you!
    – Dr. Sonnhard Graubner
    1 hour ago


















up vote
3
down vote













Since $ab+bc+ca= abc$ and $$ab+bc+cageq 3sqrt[3]{a^2b^2c^2}implies abcgeq 27$$
Now $$(a+1)(b+1)(c+1)=2abc+a+b+c+1geq 55+3sqrt[3]{27} = 64$$






share|cite|improve this answer





















  • Yup, that first passage was the one I didn't see, I'm actually terrible at AM-GM!...
    – Spasoje Durovic
    1 hour ago










  • Yeah, I've started working on inequalities about a week ago, hope I'll get better soon! cheers (nice to see you're from the Balkans too :P)
    – Spasoje Durovic
    1 hour ago












  • Actualy I'm not.
    – greedoid
    1 hour ago










  • My bad! I supposed it since you used Cyrillic
    – Spasoje Durovic
    1 hour ago






  • 1




    Never mind, it is OK
    – greedoid
    1 hour ago


















up vote
1
down vote













We need to prove that
$$sum_{cyc}(ln(1+a)-2ln2)geq0$$ or
$$sum_{cyc}left(ln(1+a)-2ln2+frac{9}{4}left(frac{1}{a}-frac{1}{3}right)right)geq0,$$
which is true because
$$f(a)=ln(1+a)-2ln2+frac{9}{4}left(frac{1}{a}-frac{1}{3}right)geq0$$ for all $a>0$.



Indeed, $$f'(a)=frac{(a-3)(4a+3)}{4a^2(a+1)},$$ which gives $a_{min}=3$ and since $f(3)=0$, we are done!






share|cite|improve this answer





















  • Can you elaborate on how you got the first expression?
    – YiFan
    25 mins ago











Your Answer





StackExchange.ifUsing("editor", function () {
return StackExchange.using("mathjaxEditing", function () {
StackExchange.MarkdownEditor.creationCallbacks.add(function (editor, postfix) {
StackExchange.mathjaxEditing.prepareWmdForMathJax(editor, postfix, [["$", "$"], ["\\(","\\)"]]);
});
});
}, "mathjax-editing");

StackExchange.ready(function() {
var channelOptions = {
tags: "".split(" "),
id: "69"
};
initTagRenderer("".split(" "), "".split(" "), channelOptions);

StackExchange.using("externalEditor", function() {
// Have to fire editor after snippets, if snippets enabled
if (StackExchange.settings.snippets.snippetsEnabled) {
StackExchange.using("snippets", function() {
createEditor();
});
}
else {
createEditor();
}
});

function createEditor() {
StackExchange.prepareEditor({
heartbeatType: 'answer',
convertImagesToLinks: true,
noModals: true,
showLowRepImageUploadWarning: true,
reputationToPostImages: 10,
bindNavPrevention: true,
postfix: "",
imageUploader: {
brandingHtml: "Powered by u003ca class="icon-imgur-white" href="https://imgur.com/"u003eu003c/au003e",
contentPolicyHtml: "User contributions licensed under u003ca href="https://creativecommons.org/licenses/by-sa/3.0/"u003ecc by-sa 3.0 with attribution requiredu003c/au003e u003ca href="https://stackoverflow.com/legal/content-policy"u003e(content policy)u003c/au003e",
allowUrls: true
},
noCode: true, onDemand: true,
discardSelector: ".discard-answer"
,immediatelyShowMarkdownHelp:true
});


}
});














draft saved

draft discarded


















StackExchange.ready(
function () {
StackExchange.openid.initPostLogin('.new-post-login', 'https%3a%2f%2fmath.stackexchange.com%2fquestions%2f3026163%2fproving-a-three-variables-inequality%23new-answer', 'question_page');
}
);

Post as a guest















Required, but never shown

























4 Answers
4






active

oldest

votes








4 Answers
4






active

oldest

votes









active

oldest

votes






active

oldest

votes








up vote
7
down vote



accepted










By the super-additivity of the geometric mean / convexity of $log(e^x+1)$
$$ (1+a)(1+b)(1+c) geq (1+sqrt[3]{abc})^3 $$
and by the GM-HM inequality
$$ sqrt[3]{abc} geq frac{3}{frac{1}{a}+frac{1}{b}+frac{1}{c}}=3.$$






share|cite|improve this answer

















  • 1




    Please explain the downvote. I do not see anything wrong with this approach, it is pretty straightforward.
    – Jack D'Aurizio
    2 hours ago






  • 3




    Maybe the downvoter's arithmetic is so bad that they want you explain that $4^3 = 64$ $ddot{smile}$.
    – Rob Arthan
    2 hours ago








  • 1




    @JackD'Aurizio I'm convinced that there are users who stays here on MSE only to downvote, probably to give vent to frustration in the real world!
    – gimusi
    2 hours ago








  • 1




    @greedoid Nothing personal with the comment, I was only joking of course.
    – gimusi
    2 hours ago






  • 1




    @RobArthan Not a theory but only a joke! Bye :)
    – gimusi
    1 hour ago















up vote
7
down vote



accepted










By the super-additivity of the geometric mean / convexity of $log(e^x+1)$
$$ (1+a)(1+b)(1+c) geq (1+sqrt[3]{abc})^3 $$
and by the GM-HM inequality
$$ sqrt[3]{abc} geq frac{3}{frac{1}{a}+frac{1}{b}+frac{1}{c}}=3.$$






share|cite|improve this answer

















  • 1




    Please explain the downvote. I do not see anything wrong with this approach, it is pretty straightforward.
    – Jack D'Aurizio
    2 hours ago






  • 3




    Maybe the downvoter's arithmetic is so bad that they want you explain that $4^3 = 64$ $ddot{smile}$.
    – Rob Arthan
    2 hours ago








  • 1




    @JackD'Aurizio I'm convinced that there are users who stays here on MSE only to downvote, probably to give vent to frustration in the real world!
    – gimusi
    2 hours ago








  • 1




    @greedoid Nothing personal with the comment, I was only joking of course.
    – gimusi
    2 hours ago






  • 1




    @RobArthan Not a theory but only a joke! Bye :)
    – gimusi
    1 hour ago













up vote
7
down vote



accepted







up vote
7
down vote



accepted






By the super-additivity of the geometric mean / convexity of $log(e^x+1)$
$$ (1+a)(1+b)(1+c) geq (1+sqrt[3]{abc})^3 $$
and by the GM-HM inequality
$$ sqrt[3]{abc} geq frac{3}{frac{1}{a}+frac{1}{b}+frac{1}{c}}=3.$$






share|cite|improve this answer












By the super-additivity of the geometric mean / convexity of $log(e^x+1)$
$$ (1+a)(1+b)(1+c) geq (1+sqrt[3]{abc})^3 $$
and by the GM-HM inequality
$$ sqrt[3]{abc} geq frac{3}{frac{1}{a}+frac{1}{b}+frac{1}{c}}=3.$$







share|cite|improve this answer












share|cite|improve this answer



share|cite|improve this answer










answered 2 hours ago









Jack D'Aurizio

284k33275653




284k33275653








  • 1




    Please explain the downvote. I do not see anything wrong with this approach, it is pretty straightforward.
    – Jack D'Aurizio
    2 hours ago






  • 3




    Maybe the downvoter's arithmetic is so bad that they want you explain that $4^3 = 64$ $ddot{smile}$.
    – Rob Arthan
    2 hours ago








  • 1




    @JackD'Aurizio I'm convinced that there are users who stays here on MSE only to downvote, probably to give vent to frustration in the real world!
    – gimusi
    2 hours ago








  • 1




    @greedoid Nothing personal with the comment, I was only joking of course.
    – gimusi
    2 hours ago






  • 1




    @RobArthan Not a theory but only a joke! Bye :)
    – gimusi
    1 hour ago














  • 1




    Please explain the downvote. I do not see anything wrong with this approach, it is pretty straightforward.
    – Jack D'Aurizio
    2 hours ago






  • 3




    Maybe the downvoter's arithmetic is so bad that they want you explain that $4^3 = 64$ $ddot{smile}$.
    – Rob Arthan
    2 hours ago








  • 1




    @JackD'Aurizio I'm convinced that there are users who stays here on MSE only to downvote, probably to give vent to frustration in the real world!
    – gimusi
    2 hours ago








  • 1




    @greedoid Nothing personal with the comment, I was only joking of course.
    – gimusi
    2 hours ago






  • 1




    @RobArthan Not a theory but only a joke! Bye :)
    – gimusi
    1 hour ago








1




1




Please explain the downvote. I do not see anything wrong with this approach, it is pretty straightforward.
– Jack D'Aurizio
2 hours ago




Please explain the downvote. I do not see anything wrong with this approach, it is pretty straightforward.
– Jack D'Aurizio
2 hours ago




3




3




Maybe the downvoter's arithmetic is so bad that they want you explain that $4^3 = 64$ $ddot{smile}$.
– Rob Arthan
2 hours ago






Maybe the downvoter's arithmetic is so bad that they want you explain that $4^3 = 64$ $ddot{smile}$.
– Rob Arthan
2 hours ago






1




1




@JackD'Aurizio I'm convinced that there are users who stays here on MSE only to downvote, probably to give vent to frustration in the real world!
– gimusi
2 hours ago






@JackD'Aurizio I'm convinced that there are users who stays here on MSE only to downvote, probably to give vent to frustration in the real world!
– gimusi
2 hours ago






1




1




@greedoid Nothing personal with the comment, I was only joking of course.
– gimusi
2 hours ago




@greedoid Nothing personal with the comment, I was only joking of course.
– gimusi
2 hours ago




1




1




@RobArthan Not a theory but only a joke! Bye :)
– gimusi
1 hour ago




@RobArthan Not a theory but only a joke! Bye :)
– gimusi
1 hour ago










up vote
5
down vote













I think you mean $$a,b,c>0$$ in this case we have
$$frac{a+b+c}{3}geq frac{3}{frac{1}{a}+frac{1}{b}+frac{1}{c}}$$ so $$a+b+cgeq 9$$ and we get also $$frac{bc+ac+ab}{3}geq sqrt[3]{(abc)^2}$$ so $$abcgeq 27$$ and $$ab+ac+bcgeq 27$$ putting things together we have
$$(a+1)(b+1)(c+1)=abc+ab+bc+ca+a+b+c+1geq 27+27+10=64$$






share|cite|improve this answer





















  • Welcome back Dr. Graubner!
    – gimusi
    1 hour ago










  • Yeah, OP forgot to mention that $a,b,c$ have to be positive, I forgot to add that I had noticed that
    – Spasoje Durovic
    1 hour ago










  • Thanks, i'm also glad to see you!
    – Dr. Sonnhard Graubner
    1 hour ago















up vote
5
down vote













I think you mean $$a,b,c>0$$ in this case we have
$$frac{a+b+c}{3}geq frac{3}{frac{1}{a}+frac{1}{b}+frac{1}{c}}$$ so $$a+b+cgeq 9$$ and we get also $$frac{bc+ac+ab}{3}geq sqrt[3]{(abc)^2}$$ so $$abcgeq 27$$ and $$ab+ac+bcgeq 27$$ putting things together we have
$$(a+1)(b+1)(c+1)=abc+ab+bc+ca+a+b+c+1geq 27+27+10=64$$






share|cite|improve this answer





















  • Welcome back Dr. Graubner!
    – gimusi
    1 hour ago










  • Yeah, OP forgot to mention that $a,b,c$ have to be positive, I forgot to add that I had noticed that
    – Spasoje Durovic
    1 hour ago










  • Thanks, i'm also glad to see you!
    – Dr. Sonnhard Graubner
    1 hour ago













up vote
5
down vote










up vote
5
down vote









I think you mean $$a,b,c>0$$ in this case we have
$$frac{a+b+c}{3}geq frac{3}{frac{1}{a}+frac{1}{b}+frac{1}{c}}$$ so $$a+b+cgeq 9$$ and we get also $$frac{bc+ac+ab}{3}geq sqrt[3]{(abc)^2}$$ so $$abcgeq 27$$ and $$ab+ac+bcgeq 27$$ putting things together we have
$$(a+1)(b+1)(c+1)=abc+ab+bc+ca+a+b+c+1geq 27+27+10=64$$






share|cite|improve this answer












I think you mean $$a,b,c>0$$ in this case we have
$$frac{a+b+c}{3}geq frac{3}{frac{1}{a}+frac{1}{b}+frac{1}{c}}$$ so $$a+b+cgeq 9$$ and we get also $$frac{bc+ac+ab}{3}geq sqrt[3]{(abc)^2}$$ so $$abcgeq 27$$ and $$ab+ac+bcgeq 27$$ putting things together we have
$$(a+1)(b+1)(c+1)=abc+ab+bc+ca+a+b+c+1geq 27+27+10=64$$







share|cite|improve this answer












share|cite|improve this answer



share|cite|improve this answer










answered 2 hours ago









Dr. Sonnhard Graubner

71.6k32864




71.6k32864












  • Welcome back Dr. Graubner!
    – gimusi
    1 hour ago










  • Yeah, OP forgot to mention that $a,b,c$ have to be positive, I forgot to add that I had noticed that
    – Spasoje Durovic
    1 hour ago










  • Thanks, i'm also glad to see you!
    – Dr. Sonnhard Graubner
    1 hour ago


















  • Welcome back Dr. Graubner!
    – gimusi
    1 hour ago










  • Yeah, OP forgot to mention that $a,b,c$ have to be positive, I forgot to add that I had noticed that
    – Spasoje Durovic
    1 hour ago










  • Thanks, i'm also glad to see you!
    – Dr. Sonnhard Graubner
    1 hour ago
















Welcome back Dr. Graubner!
– gimusi
1 hour ago




Welcome back Dr. Graubner!
– gimusi
1 hour ago












Yeah, OP forgot to mention that $a,b,c$ have to be positive, I forgot to add that I had noticed that
– Spasoje Durovic
1 hour ago




Yeah, OP forgot to mention that $a,b,c$ have to be positive, I forgot to add that I had noticed that
– Spasoje Durovic
1 hour ago












Thanks, i'm also glad to see you!
– Dr. Sonnhard Graubner
1 hour ago




Thanks, i'm also glad to see you!
– Dr. Sonnhard Graubner
1 hour ago










up vote
3
down vote













Since $ab+bc+ca= abc$ and $$ab+bc+cageq 3sqrt[3]{a^2b^2c^2}implies abcgeq 27$$
Now $$(a+1)(b+1)(c+1)=2abc+a+b+c+1geq 55+3sqrt[3]{27} = 64$$






share|cite|improve this answer





















  • Yup, that first passage was the one I didn't see, I'm actually terrible at AM-GM!...
    – Spasoje Durovic
    1 hour ago










  • Yeah, I've started working on inequalities about a week ago, hope I'll get better soon! cheers (nice to see you're from the Balkans too :P)
    – Spasoje Durovic
    1 hour ago












  • Actualy I'm not.
    – greedoid
    1 hour ago










  • My bad! I supposed it since you used Cyrillic
    – Spasoje Durovic
    1 hour ago






  • 1




    Never mind, it is OK
    – greedoid
    1 hour ago















up vote
3
down vote













Since $ab+bc+ca= abc$ and $$ab+bc+cageq 3sqrt[3]{a^2b^2c^2}implies abcgeq 27$$
Now $$(a+1)(b+1)(c+1)=2abc+a+b+c+1geq 55+3sqrt[3]{27} = 64$$






share|cite|improve this answer





















  • Yup, that first passage was the one I didn't see, I'm actually terrible at AM-GM!...
    – Spasoje Durovic
    1 hour ago










  • Yeah, I've started working on inequalities about a week ago, hope I'll get better soon! cheers (nice to see you're from the Balkans too :P)
    – Spasoje Durovic
    1 hour ago












  • Actualy I'm not.
    – greedoid
    1 hour ago










  • My bad! I supposed it since you used Cyrillic
    – Spasoje Durovic
    1 hour ago






  • 1




    Never mind, it is OK
    – greedoid
    1 hour ago













up vote
3
down vote










up vote
3
down vote









Since $ab+bc+ca= abc$ and $$ab+bc+cageq 3sqrt[3]{a^2b^2c^2}implies abcgeq 27$$
Now $$(a+1)(b+1)(c+1)=2abc+a+b+c+1geq 55+3sqrt[3]{27} = 64$$






share|cite|improve this answer












Since $ab+bc+ca= abc$ and $$ab+bc+cageq 3sqrt[3]{a^2b^2c^2}implies abcgeq 27$$
Now $$(a+1)(b+1)(c+1)=2abc+a+b+c+1geq 55+3sqrt[3]{27} = 64$$







share|cite|improve this answer












share|cite|improve this answer



share|cite|improve this answer










answered 2 hours ago









greedoid

35.8k114590




35.8k114590












  • Yup, that first passage was the one I didn't see, I'm actually terrible at AM-GM!...
    – Spasoje Durovic
    1 hour ago










  • Yeah, I've started working on inequalities about a week ago, hope I'll get better soon! cheers (nice to see you're from the Balkans too :P)
    – Spasoje Durovic
    1 hour ago












  • Actualy I'm not.
    – greedoid
    1 hour ago










  • My bad! I supposed it since you used Cyrillic
    – Spasoje Durovic
    1 hour ago






  • 1




    Never mind, it is OK
    – greedoid
    1 hour ago


















  • Yup, that first passage was the one I didn't see, I'm actually terrible at AM-GM!...
    – Spasoje Durovic
    1 hour ago










  • Yeah, I've started working on inequalities about a week ago, hope I'll get better soon! cheers (nice to see you're from the Balkans too :P)
    – Spasoje Durovic
    1 hour ago












  • Actualy I'm not.
    – greedoid
    1 hour ago










  • My bad! I supposed it since you used Cyrillic
    – Spasoje Durovic
    1 hour ago






  • 1




    Never mind, it is OK
    – greedoid
    1 hour ago
















Yup, that first passage was the one I didn't see, I'm actually terrible at AM-GM!...
– Spasoje Durovic
1 hour ago




Yup, that first passage was the one I didn't see, I'm actually terrible at AM-GM!...
– Spasoje Durovic
1 hour ago












Yeah, I've started working on inequalities about a week ago, hope I'll get better soon! cheers (nice to see you're from the Balkans too :P)
– Spasoje Durovic
1 hour ago






Yeah, I've started working on inequalities about a week ago, hope I'll get better soon! cheers (nice to see you're from the Balkans too :P)
– Spasoje Durovic
1 hour ago














Actualy I'm not.
– greedoid
1 hour ago




Actualy I'm not.
– greedoid
1 hour ago












My bad! I supposed it since you used Cyrillic
– Spasoje Durovic
1 hour ago




My bad! I supposed it since you used Cyrillic
– Spasoje Durovic
1 hour ago




1




1




Never mind, it is OK
– greedoid
1 hour ago




Never mind, it is OK
– greedoid
1 hour ago










up vote
1
down vote













We need to prove that
$$sum_{cyc}(ln(1+a)-2ln2)geq0$$ or
$$sum_{cyc}left(ln(1+a)-2ln2+frac{9}{4}left(frac{1}{a}-frac{1}{3}right)right)geq0,$$
which is true because
$$f(a)=ln(1+a)-2ln2+frac{9}{4}left(frac{1}{a}-frac{1}{3}right)geq0$$ for all $a>0$.



Indeed, $$f'(a)=frac{(a-3)(4a+3)}{4a^2(a+1)},$$ which gives $a_{min}=3$ and since $f(3)=0$, we are done!






share|cite|improve this answer





















  • Can you elaborate on how you got the first expression?
    – YiFan
    25 mins ago















up vote
1
down vote













We need to prove that
$$sum_{cyc}(ln(1+a)-2ln2)geq0$$ or
$$sum_{cyc}left(ln(1+a)-2ln2+frac{9}{4}left(frac{1}{a}-frac{1}{3}right)right)geq0,$$
which is true because
$$f(a)=ln(1+a)-2ln2+frac{9}{4}left(frac{1}{a}-frac{1}{3}right)geq0$$ for all $a>0$.



Indeed, $$f'(a)=frac{(a-3)(4a+3)}{4a^2(a+1)},$$ which gives $a_{min}=3$ and since $f(3)=0$, we are done!






share|cite|improve this answer





















  • Can you elaborate on how you got the first expression?
    – YiFan
    25 mins ago













up vote
1
down vote










up vote
1
down vote









We need to prove that
$$sum_{cyc}(ln(1+a)-2ln2)geq0$$ or
$$sum_{cyc}left(ln(1+a)-2ln2+frac{9}{4}left(frac{1}{a}-frac{1}{3}right)right)geq0,$$
which is true because
$$f(a)=ln(1+a)-2ln2+frac{9}{4}left(frac{1}{a}-frac{1}{3}right)geq0$$ for all $a>0$.



Indeed, $$f'(a)=frac{(a-3)(4a+3)}{4a^2(a+1)},$$ which gives $a_{min}=3$ and since $f(3)=0$, we are done!






share|cite|improve this answer












We need to prove that
$$sum_{cyc}(ln(1+a)-2ln2)geq0$$ or
$$sum_{cyc}left(ln(1+a)-2ln2+frac{9}{4}left(frac{1}{a}-frac{1}{3}right)right)geq0,$$
which is true because
$$f(a)=ln(1+a)-2ln2+frac{9}{4}left(frac{1}{a}-frac{1}{3}right)geq0$$ for all $a>0$.



Indeed, $$f'(a)=frac{(a-3)(4a+3)}{4a^2(a+1)},$$ which gives $a_{min}=3$ and since $f(3)=0$, we are done!







share|cite|improve this answer












share|cite|improve this answer



share|cite|improve this answer










answered 1 hour ago









Michael Rozenberg

94.6k1588183




94.6k1588183












  • Can you elaborate on how you got the first expression?
    – YiFan
    25 mins ago


















  • Can you elaborate on how you got the first expression?
    – YiFan
    25 mins ago
















Can you elaborate on how you got the first expression?
– YiFan
25 mins ago




Can you elaborate on how you got the first expression?
– YiFan
25 mins ago


















draft saved

draft discarded




















































Thanks for contributing an answer to Mathematics Stack Exchange!


  • Please be sure to answer the question. Provide details and share your research!

But avoid



  • Asking for help, clarification, or responding to other answers.

  • Making statements based on opinion; back them up with references or personal experience.


Use MathJax to format equations. MathJax reference.


To learn more, see our tips on writing great answers.





Some of your past answers have not been well-received, and you're in danger of being blocked from answering.


Please pay close attention to the following guidance:


  • Please be sure to answer the question. Provide details and share your research!

But avoid



  • Asking for help, clarification, or responding to other answers.

  • Making statements based on opinion; back them up with references or personal experience.


To learn more, see our tips on writing great answers.




draft saved


draft discarded














StackExchange.ready(
function () {
StackExchange.openid.initPostLogin('.new-post-login', 'https%3a%2f%2fmath.stackexchange.com%2fquestions%2f3026163%2fproving-a-three-variables-inequality%23new-answer', 'question_page');
}
);

Post as a guest















Required, but never shown





















































Required, but never shown














Required, but never shown












Required, but never shown







Required, but never shown

































Required, but never shown














Required, but never shown












Required, but never shown







Required, but never shown







Popular posts from this blog

404 Error Contact Form 7 ajax form submitting

How to know if a Active Directory user can login interactively

TypeError: fit_transform() missing 1 required positional argument: 'X'